Is (B.\nabla)A the same as B(\nabla.A)?

Click For Summary
The discussion revolves around proving the vector identity involving the gradient, divergence, and curl of vector fields A and B. The left-hand side (LHS) of the equation needs to be expanded and compared to the right-hand side (RHS) to establish equality. A key point of confusion is whether (B·∇)A is equivalent to B(∇·A), which is clarified as incorrect since it would lead to cancellation of components. The participant suggests using the product rule for gradients to handle the components properly. The discussion emphasizes the importance of analyzing individual components of the vector expressions to verify the identity.
featheredteap
Messages
4
Reaction score
0

Homework Statement



Prove the following vector identity:

\nablax(AxB) = (B.\nabla)A - (A.\nabla)B + A(\nabla.B) - B(\nabla.A)

Where A and B are vector fields.

Homework Equations



Curl, divergence, gradient

The Attempt at a Solution



I think I know how to do this: I have to expand out the LHS and the RHS and show that they equal one another. To do this I need to use the product rule when taking the gradient of components with more than one term multiplied together.

What I don't understand is what's going on on the RHS: doesn't (B.\nabla)A = B(\nabla.A) ? (Obviously this can't be the case since then all the components would cancel to zero.) So how does this really work?
 
Last edited:
Physics news on Phys.org
No, since if
<br /> \begin{displaymath}<br /> \begin{array}{rcl}<br /> \mathbf{A} &amp; = &amp; A_{x}\mathbf{i}+A_{x}\mathbf{j}+A_{z}\mathbf{k} \\<br /> \mathbf{B} &amp; = &amp; B_{x}\mathbf{i}+B_{x}\mathbf{j}+B_{z}\mathbf{k}<br /> \end{array}<br /> \end{displaymath}<br />
Then:
<br /> \begin{displaymath}<br /> \begin{array}{rcl}<br /> \mathbf{B}\cdot\nabla &amp; = &amp; B_{x}\partial_{x}+B_{y}\partial_{y}+B_{z}\partial_{z} \\<br /> \nabla\cdot\mathbf{A} &amp; = &amp; \partial_{x}A_{x}+\partial_{y}A_{y}+\partial_{z}A_{z}<br /> \end{array}<br /> \end{displaymath}<br />
Using the above compare one component of the two vector expressions and see if they're the same.
 
Question: A clock's minute hand has length 4 and its hour hand has length 3. What is the distance between the tips at the moment when it is increasing most rapidly?(Putnam Exam Question) Answer: Making assumption that both the hands moves at constant angular velocities, the answer is ## \sqrt{7} .## But don't you think this assumption is somewhat doubtful and wrong?

Similar threads

  • · Replies 10 ·
Replies
10
Views
4K
  • · Replies 9 ·
Replies
9
Views
2K
  • · Replies 9 ·
Replies
9
Views
2K
  • · Replies 5 ·
Replies
5
Views
2K
  • · Replies 6 ·
Replies
6
Views
2K
  • · Replies 3 ·
Replies
3
Views
2K
  • · Replies 2 ·
Replies
2
Views
4K
Replies
1
Views
1K
  • · Replies 2 ·
Replies
2
Views
3K
  • · Replies 4 ·
Replies
4
Views
2K